ChaseDream
搜索
返回列表 发新帖
楼主: laserlee1
打印 上一主题 下一主题

LSAT CR(9702) 真题详解(官方解释)

[复制链接]
11#
发表于 2005-7-20 13:39:00 | 只看该作者
awesome stuff..thanks a bunch

12#
发表于 2005-10-9 08:57:00 | 只看该作者

The Official LSAT PrepTest With Explanations Volume 1(有点像解题方法)


是这本书,无疑,但是还不全!楼主能补全么?谢谢。

13#
发表于 2005-12-27 16:47:00 | 只看该作者


衷心感谢楼主发布,但9702还有一个 LR Section。下面的内容是我从网上找到的,相信就是这个Section的题目与官方解释。需要说明两点,一是我找到的也不全,仅十三题,二是惭愧自己是电脑菜鸟,只会粘贴不会以文档形式发布资源。希望下面的内容对大家有用,也希望有热心人将这个Section所缺的题目补足。



1. Historian: Megapolis was once administered wisely, with vision and with universal public support. That’s how it became a great city.


City council member: Not so. Megapolis' past glory was achieved not because of greater vision, but because power was concentrated in the hands of autocrats and hence more easily exercised.


Which one of the following best captures what is at issue in the dispute between the historian and the city council member?


(A) whether Megapolis was once a great city


(B) what the best explanation is for Megapolis' past glory


(C) whether vision and wisdom are important in urban administration


(D) whether the administration of Megapolis once had popular support


(E) why Megapolis is no longer a well-run city



General Description: This question requires you to identify the issue over which the historian and the city council member disagree.



A. Incorrect. Both the historian ("it became a great city") and the city council member ("Megapolis' past glory") believe that Megapolis was once a great city.



B. Correct. The historian gives an explanation of how Megapolis became a great city: it was "administered wisely, with vision and with universal public support." The city council member responds with "Not so," and goes on to give a different explanation of Megapolis' greatness. This is the best answer.



C. Incorrect. Neither the historian nor the city council member takes a stand on this general issue.



D. Incorrect. The historian makes a statement to this effect, but it is never denied by the city council member.



E. Incorrect. The current status of Megapolis is not discussed.



Difficulty Level: Easy


Tips and Pitfalls: In a "point of disagreement" question, a choice that presents a claim that is supported or denied by one speaker, but about which the other speaker has made no commitment either way, cannot be correct. In other words, for a response to be correct, it is not sufficient that the speakers might disagree about it. Limit your analysis to what is actually said; avoid the temptation to speculate about what (else) either speaker might believe.



2. The striking similarities between Greek temples and subsequent Roman villas testify to the fact that great architects typically draw inspiration from the work of other architects. Such drawing of inspiration does not mean, however, a slavish lack of originality; building according to formulas does not make good buildings. When formulas are used, the results are either bland or disastrous. By contrast, a great architect creates fresh architectural compositions out of familiar forms and synthesizes past and present into a new, richer whole.


Which one of the following can be inferred from the passage?


(A) Roman architects designed many buildings with little or no originality of thought.


(B) The essence of good architecture is faithful reproduction of established models.


(C) Buildings with unfamiliar forms are likely to be either bland or architectural disasters.


(D) Some Roman architecture exemplifies the creation of fresh architectural compositions out of familiar forms.


(E) Some Greek temples were not themselves inspired by older models.



General Description: This question asks you to draw a logical inference from the information given in the passage.



A. Incorrect. While this statement may be true, the passage provides no support for it. On the contrary, the passage is about great architects, who (according to the passage) do not exhibit "a slavish lack of originality."



B. Incorrect. If anything, the passage supports the opposite conclusion, in saying that "building according to formulas does not make good buildings."



C. Incorrect. The passage does not address the potential results of following unfamiliar forms in creating buildings.



D. Correct. The passage says that the similarities between Roman villas and earlier Greek temples show that great architects can be inspired by other architects. Thus, the passage implies that some Roman architects were great. Then the passage says that a great architect "creates fresh architectural compositions out of familiar forms." So it follows that some Roman architecture-the architecture created by the great Roman architects--exemplifies the creation of fresh architectural compositions out of familiar forms.



E. Incorrect. The architectural features of Greek temples are not discussed in the passage; neither is the source of Greek architectural inspiration discussed.



Difficulty Level: Easy


Tips and Pitfalls: Read questions carefully. If a question asks which statement is supported by the passage, do not choose a response just because it seems on independent grounds to be plausible or likely you must pick a response based on the information given in the passage.



3. About two million years ago, lava dammed up a river in western Asia and caused a small lake to form. The lake existed for about half a million years. Bones of an early human ancestor were recently found in the ancient lake-bottom sediments that lie on top of the layer of lava. Therefore, ancestors of modern human lived in western Asia between two million and one-and-a-half million years ago.


Which one of the following is an assumption required by the argument?


(A) There ere no other lakes in the immediate area before the lava dammed up the river.


(B) The lake contained fish that the human ancestors could have used for food.


(C) The lava that lay under the lake-bottom sediments did not contain any human fossil remains.


(D) The lake was deep enough that a person could drown in it.


(E) The bones were already in the sediments by the time the lake dried up.



General Description: This question asks you to find the assumption required by the argument. In other words, find the statement whose truth is required if the argument is to succeed in demonstrating its conclusion.



A. Incorrect. The existence of other lakes in the area is irrelevant to the argument.



B. Incorrect. If response (B) turned out to be true, that might provide a reason why humans were in the area of the lake, but this particular explanation need not be assumed in order for the argument to succeed in demonstrating its conclusion.



C. Incorrect. It does not matter for the argument whether or not there were such remains in the lava, and the argument does not state or imply that there were no humans in the region prior to two million years ago. This was by far the most popular incorrect response.



D. Incorrect. The remains could have gotten into the lake in any number of other ways; to give just one, perhaps the people in the area put their dead into the lake.



E. Correct. If the bones were not already in the sediments when the lake dried up, that means that they got into the sediments later; that is, less than one-and-a-half million years ago. But then their existence would not provide evidence that there were human ancestors in western Asia between two million and one-and-a-half million years ago; that is, the conclusion of the argument would not follow if (E) is false.



Difficulty Level: Medium difficulty


Tips and Pitfalls: Another way to think about the question of whether an assumption is required by an argument is to think about what happens to the argument if the assumption turns out to be false. If the argument cannot possibly succeed when the assumption is false, then the assumption is required by the argument. Also, when asked for a necessary assumption, be careful not to pick an assumption that is stronger or broader than what is strictly necessary for the argument to succeed, even if making that assumption might strengthen the argument.



4. A few people who are bad writers simply cannot improve their writing, whether or not they receive instruction. Still, most bad writers can at least be taught to improve their writing enough so that they are no longer bad writers. However, no one can become a great writer simply by being taught how to be a better writer, since great writers must have not just skill, but also talent.


Which one of the following can be properly inferred from the passage above?


(A) All bad writers can become better writers.


(B) All great writers had to be taught to become better writers.


(C) Some bad writers can never become great writers.


(D) Some bad writers can become great writers.


(E) Some great writers can be taught to be even better writers.



General Description: This question asks you to find the response that can be inferred from the given passage. A statement that may well be true, but that is irrelevant to the passage, cannot be the best answer. Even a response that presents information consistent with the passage need not be the best answer. Rather, the passage must provide grounds or support for inferring the response in order for that response to be the best answer.



A. Incorrect. This statement is contradicted by the first sentence in the passage, which states that there are some bad writers who cannot become better writers.



B. Incorrect. This statement is consistent with the passage. The passage allows the possibility that all great writers had to be taught to become better writers, though it says (in the last sentence) that teaching is not sufficient for becoming a great writer. But the passage also allows the possibility that some great writers did not have to be taught to become better writers. Since both the statement and its opposite are consistent with the passage, the statement in response (B) cannot be inferred from the passage.



C. Correct. This follows from the first sentence: Since some people are bad writers who cannot improve their writing, and since bad writers would have to improve their writing as a precondition for becoming great writers, it follows that there are some people who are bad writers and who can never become great writers.



D. Incorrect. Given the passage, there may well be some bad writers who can become great writers, but it might be instead that no bad writers can become great writers. Since both the statement and its opposite are consistent with the passage, the statement in response (D) cannot be inferred from the passage.



E. Incorrect. The passage makes no claims about how a great writer could become an even better writer, so this response cannot be inferred from the passage.



Difficulty Level: Medium difficulty


Tips and Pitfalls: If a question asks what can be properly inferred from the passage, do not choose a response simply because it is consistent with the passage. The correct response will be one that follows logically from the statements in the passage; that is, the one that is consistent with the passage, but whose opposite is not consistent with the passage.



Question 5-6


Paretan newspaper editor: David Salino assails as distorted our quotation of remarks on Paretan values by the Qinkoan prime minister and bemoans what he sees as the likely consequences for Paretan-Qinkoan relations. Those consequences will not be our fault, however, since officials at the Qinkoan embassy and scholars at the Institute of Qinkoan Studies have all confirmed that, as printed, our quotation was an acceptable translation of the prime minister's remarks: "No newspaper can fairly be blamed for the consequences of its reporting when that reporting is accurate.



5. Which one of the following most accurately expresses the conclusion in the editor's response to David Salino?


(A) What the prime minister said about Paretan values is not a distortion.


(B) Assessing the likely consequences of reporting the prime minister's remarks is irrelevant to the question of whether they ought to have been reported.


(C) The newspaper's rendering of the prime minister's remarks was not inaccurate reporting according to the authorities who are in the best position to judge.


(D) The newspaper cannot be held responsible for the adverse consequences that David Salino claims will likely result from the quotation of the prime minister's remarks in the newspaper.


(E) David Salino's assessment of the likely consequences of reporting the prime minister's remarks is inaccurate.



General Description: This question asks you to identify the conclusion of an argument. To answer this sort of question, you must be able to distinguish the conclusion from other parts of an argument (such as its premises), as well as from other parts of the passage that may not be part of the argument at all.



A. Incorrect. The question of the accuracy of the prime minister's remarks does not arise in the passage. To be sure, the issue of the accuracy of the newspaper's translation of those remarks is discussed, but that is a different issue-and that discussion is not the conclusion of the editor's argument.



B. Incorrect. The question of whether the remarks ought to have been reported arises at best obliquely in the passage, and the passage does not discuss at all what factors might be relevant or irrelevant to that question.



C. Incorrect. Though this claim is made by the newspaper editor (in the second sentence), it is presented as evidence in support of the conclusion (that is, as a premise), not as the conclusion itself.



D. Correct. The newspaper editor's conclusion is stated in the passage as "[t]hose consequences will not be our fault," after which the evidence that is meant to support that conclusion is presented. This is the best answer.



E. Incorrect. The passage takes no stand on whether Salino's predictions about consequences are accurate or not; the passage says only what will be the case if those consequences actually come to pass.



Difficulty Level: Medium difficulty


Tips and Pitfalls: In identifying an argument's conclusion, be careful to pick neither a statement that makes up part of the argument's evidence or premises, nor a statement that seems plausible, given the passage, but is not the actual conclusion drawn by the argument in the passage



6. Which one of the following is an assumption on which the editor's argument depends?


(A) The confirmation that the translation is acceptable is sufficient to show that the prime minister's remarks were accurately reported.


(B) Newspapers ought not to consider the consequences of their coverage in deciding what to report.


(C) If the newspapers' rendering of the prime minister's remarks was not distorted, then there is no reason to fear adverse consequences from reporting the remarks.


(D) If David Salino was prepared to praise the newspaper for any favorable consequences of quoting the prime minister's remarks, he could then hold the newspaper to blame for adverse consequences.


(E) Only scholars or people with official standing are in a position to pass judgment on whether a translation of Qinkoan into Paretan is acceptable.



General Description: This question asks you to find an assumption on which the editor's argument depends. In other words, find the statement whose truth is required if the argument is to succeed in demonstrating its conclusion.



A. Correct. The editor presents two premises in support of the conclusion that the newspaper cannot be blamed for the consequences of its reporting: First, that the quotation was an acceptable translation, and second, that "no newspaper can fairly be blamed for the consequences of its reporting when that reporting is accurate." For this second premise to have relevance to the first, it must be the case that reporting the acceptable translation was in fact reporting accurately; that is, response (A) must be assumed to be true.



B. Incorrect. The editor need not assume that newspapers should not consider the consequences of their coverage; the point of the argument is that the newspaper should not be blamed for such consequences, if the reporting was accurate.



C. Incorrect. The editor is not taking a stand on the question of whether any adverse consequences will actually occur, and so need not assume response (C). The point of the argument is just that if they do occur, the newspaper should not be blamed for them, if the newspaper's reporting was accurate.



D. Incorrect. This statement is irrelevant to the editor's argument.



E. Incorrect. The editor does presume that confirmation of a translation's accuracy by Qinkoan officials and scholars suffices to show that the translation is, in fact, accurate. But the editor does not (and need not) assume that this would be the only way to certify the accuracy of such a translation.



Difficulty Level: Medium difficulty


Tips and Pitfalls: Another way to think about the question of whether an assumption is required by an argument is to think about what happens to the argument if the assumption turns out to be false. If the argument cannot possibly succeed when the assumption is false, then the assumption is required by the argument.



Question 7-8


Automobile manufacturers who began two decades ago to design passenger vehicles that were more fuel-efficient faced a dilemma in the fact that the lighter, more efficient vehicles were less safe on high-speed highways. However, the manufacturers avoided this dilemma by producing two types of passenger vehicles: a lighter vehicle for medium-speed, local transportation, and a heavier, safer vehicle for long-distance travel. Since most automobile traffic is local, a net savings in fuel use was achieved with no loss in safety.



7. Which one of the following, if true, most strengthens the argument?


(A) Most households whose members do any long-distance driving own at least two passenger vehicles.


(B) There are more cars using high-speed highways today than there were two decades ago.


(C) Even large automobiles are lighter today than similar-sized vehicles were two decades ago.


(D) Most high-speed highways are used by both commercial vehicles and passenger vehicles.


(E) Some automobile manufacturers designed prototypes for fuel-efficient passenger vehicles more than two decades ago.



General Description: This question asks you to determine which response most strengthens the argument. In approaching such questions, you should identify the conclusion of the argument, and find the response that, if true, adds to the argument's support for its conclusion.



A. Correct. The argument says that automobile manufacturers addressed the trade-off between efficiency and safety by designing two kinds of cars, a more efficient but less safe car for local travel, and a less efficient but safer car for long-distance travel. Further, the argument says that "most automobile traffic is local," and then draws its conclusion that "a net savings in fuel use was achieved [with the two car designs] with no loss in safety." The more people actually use cars in the way that leads to greatest efficiency and safety, the stronger the argument is: That is, to the extent that people actually use the more efficient cars for local travel and the safer cars for long-distance travel, the argument is stronger. Response (A) strengthens the argument by showing that it is at least possible that most people do use their cars in that way, and is the best answer.



B. Incorrect. At best, the increased number of cars using high-speed highways is essentially irrelevant to the argument.



C. Incorrect. Without further information, it is not clear how the truth of this statement would bear on the argument. According to the passage, lighter cars are more fuel-efficient, but less safe. From response (C) alone, we cannot tell to what extent its truth would affect the net relationship between safety and fuel efficiency.



D. Incorrect. The relationship of commercial vehicles to the situation described in the passage does not arise in the argument; thus this response is, without further information, irrelevant to the argument.


E. Incorrect. This response is at best irrelevant to the argument; exactly when automobile manufacturers began designing fuel-efficient cars is not at issue.



Difficulty Level: Relatively easy


Tips and Pitfalls: In answering a question asking for strengthening evidence, be careful not to read more into a response statement than is actually there. A statement that could strengthen the argument, if other (unstated) facts obtain, is not as good an answer as a statement that strengthens the argument on its own.



8. Which one of the following, if true, most seriously weakens the argument?


(A) Lighter vehicles can have smaller, more fuel-efficient engines without sacrificing power.


(B) Long-distance drivers are usually made on high-speed highways.


(C) Every automobile manufacturer now produces a greater number of fuel-efficient passenger vehicles than that manufacturer did two decades ago


(D) Many drivers use high-speed highways even when traveling locally


(E) Passenger vehicles today weigh less, on average, than did passenger vehicles two decades ago.



General Description: This question asks you to determine which response most weakens the argument. In approaching such questions, you should identify the conclusion of the argument, and find the response that, if true, undermines the argument's support for its conclusion.



A. Incorrect. If anything, this statement would tend to strengthen the argument, by eliminating a reason people might not actually use the lighter, more efficient cars.



B. Incorrect. This statement tends to strengthen the argument, by asserting that part of what the argument takes for granted (namely, that long-distance travel tends to take place on high-speed highways) is actually true.



C. Incorrect. The number of fuel-efficient cars being produced, and the distribution of such cars among the various automobile manufacturers, is irrelevant to the argument.



D. Correct. The argument is weakened to the extent that local travel takes place on high-speed highways. The argument says that "lighter, more efficient vehicles were less safe on high-speed highways." So if people use the more efficient but lighter, less-safe cars for local travel, and much local travel takes place on high-speed highways, the argument's conclusion that there is "no loss in safety" associated with the savings in fuel use is undermined.



E. Incorrect. The average passenger vehicle weight is irrelevant to the argument.



Difficulty Level: Easy


Tips and Pitfalls: It may seem obvious to say that if the question asks for weakening evidence, one should not choose strengthening evidence, and vice versa, but this is an easy mistake to make when answering questions quickly. If a statement does not strengthen an argument, do not assume that it must therefore weaken the argument.



9. A scholar discovered an unlabeled nineteenth-century recording of someone reciting a poem written by Walt Whitman. During the nineteenth century, recordings of poetry were not made for wide commercial sale. Rather, they were made either as rare private souvenirs of the voices of famous poets or else as publicity stunts, in which actors recorded poems that were familiar to the public. Since the Whitman poem in the recording was never even published, it is likely that the voice in the recording is actually Whitman's.


The argument proceeds by


(A) offering several pieces of evidence each of which independently points to the same conclusion


(B) distinguishing a phenomenon into two subtypes and then for a particular case eliminating one of those subtypes


(C) offering a general principle and then demonstrating that the general principle is violated in a particular case


(D) showing that two apparently mutually exclusive alternatives are actually compatible with one another


(E) explaining the historical context of an incident in order to demonstrate that each of the two possible scenarios involving that incident is as likely as the other



General Description: This question asks you to find the description of the method by which the argument proceeds.



A. Incorrect. The argument does offer several pieces of evidence, but they do not point to the conclusion independently of one another. That is, taking any


one of the argument's pieces of evidence alone does not support the conclusion; it is only together that the evidence supports the conclusion.



B. Correct. The "phenomenon" is the nineteenth century practice of recording poetry for noncommercial purposes. The "two subtypes" are (i) "[those made as] rare private souvenirs of the voices of famous poets" and (ii) "[those made] as publicity stunts, in which actors recorded poems that were familiar to the public." In the "particular case" of the recording in question, the argument eliminates the second subtype by pointing out that the poem on that recording "was never even published," that is, is not a poem that would likely have been familiar to the public. Since this response most completely and accurately describes the method of the argument in the passage, it is the best answer.



C. Incorrect. There are several statements in the passage that could be construed as "general principle," but the argument does not attempt to demonstrate that any of them is violated in the particular case in question



D. Incorrect. There are "two apparently mutually exclusive alternatives" presented- namely, that a certain recording of poetry made in the nineteenth century was made either as a souvenir of the voice of a famous poet, or else as a publicity stunt in which an actor recorded a well-known poem. But the argument does not attempt to show that these two alternatives are compatible with one another.



E. Incorrect. The argument can be construed as "explaining the historical context of an incident," though it may be debatable whether a recording counts as an "incident." But even if it does, this response is incorrect, because the argument does not attempt to demonstrate that two scenarios are equally likely. Can the contrary, the argument purports to show that one scenario is more likely than the other.



Difficulty Level: Difficult


Tips and Pitfalls: In answering questions about an argument's method, be careful to compare completely the methods of support described in the responses to the argument in the passage: If a response's method does not correspond exactly to that used in the argument, that response is not the best answer.



10. All coffee drinkers in an office ought to contribute equally to the fund that pays for the office's coffee, because, although some coffee drinkers would prefer to pay for their coffee by the cup, or in some other manner, it is better if everyone who drinks the office's coffee provides the same amount of support to the fund


(A) It offers, in place of support for its conclusion, a mere restatement of that conclusion.


(B) It overlooks the possibility that what is true under certain specified conditions is not necessarily true under most conditions.


(C) It uses emotionally charged terms to characterize unfairly the position it attempts to refute.


(D) It applies a double standard whereby one group of people is judged wrong and another group judged right for engaging in similar behavior.


(E) It offers two alternatives that do not exhaust the possibilities available and then treats those alternatives as the only possible ones.



General Description: This question asks you to find the criticism to which the reasoning of the argument in the passage is most vulnerable.



A. Correct. The argument in the passage presents no real evidence for its conclusion. Saying that "it is better if everyone... provides the same amountof support" is practically equivalent to saying "[everyone] ought to contribute equally." So presenting the first as "evidence" for the second is simply to commit the error described in response (A).



B. Incorrect. The argument is not vulnerable to this criticism, because its claims are not tied to any "certain specified conditions."



C. Incorrect. There are no terms in the argument that could be characterized as "emotionally charged."



D. Incorrect. No group of people is being "judged wrong" in the argument; the argument is not vulnerable to this criticism.



E. Incorrect. It could be argued that this response is partly accurate, inasmuch as the argument presents the alternatives (i) all coffee drinkers contribute equally and (ii) coffee drinkers pay for their coffee by the cup. But the argument does not treat these two alternatives as if they exhaust the possibilities; note the reference to "some other manner."



Difficulty Level: Relatively easy


Tips and Pitfalls: In a question that asks for a criticism of an argument's reasoning, for a response to be the best answer, it is necessary, but not sufficient, that it accurately describe some part of the argument. More than this, it must describe a feature of the argument in virtue of which the argument's reasoning is vulnerable to criticism.



11. In 1987 Fabrico, a clothing manufacturer, sold to its customers a total of 2.3 million dollars worth of clothing, a 17 percent increase over 1986 sales. In January 1988, however, Fabrico closed one of its clothing factories, citing reduced demand for its products as the major reason.


Which one of the following, if true about 1987, contributes most to a resolution of the apparent discrepancy noted between the sales figures and the reason offered for closing a factory?


(A) The total worldwide dollar value of clothing sales by manufacturers to customers increased by 5 percent over sales in 1986.


(B) Fabrico's employees dropped earlier demands for a 25 percent increase in company-paid benefits and a 15 percent increase in hourly wages.


(C) Because of escalating costs for raw materials, prices charged by Fabrico for its products were on average 42 percent higher than they were in 1986.


(D) Fabrico introduced several new styles of clothing that were generally praised by fashion critics and that sold very well.


(E) Fabrico spent less on capital improvements than it did in 1986, when it added new plant capacity and new equipment.



General Description: This question asks you to find the response that would resolve the apparent discrepancy described in the passage. To answer this question, you must understand just what makes the evidence cited into an apparent discrepancy: What is the problem that the correct response will solve?



A. Incorrect. This information about the total market for clothing sales does not help resolve a discrepancy in the behavior of one particular clothing manufacturer.



B. Incorrect. Without further information, it is impossible to know how the employees' demands bear on the situation described in the passage. Even with further information, it is difficult to see how response (B) could bear on the second part of the discrepancy in the passage (closing the factory in response to reduced demand).



C. Correct. If the selling prices for Fabrico's products were 42 percent higher in 1987 than in 1986, but the total dollar amount of Fabrico's sales were only 17 percent higher in 1987 than in 1986, that suggests that Fabrico actually sold less clothing in 1987 than in 1986. Thus it is not surprising that Fabrico would close a factory in January 1988 because of reduced demand. Response (C) shows how both pieces of information in the passage could be true; that is, it resolves the discrepancy and is the best answer.



D. Incorrect. This statement could help explain the first part of the discrepancy (increased sales), but if anything, it makes the second part (closing the factory in response to reduced demand) even more mysterious.



E. Incorrect. The amount Fabrico spends on capital improvements has an impact on its overall profit, but overall profit is not at issue in the passage: The apparent discrepancy is that Fabrico has increased sales of its products and yet claims that demand for those products is reduced.



Difficulty Level: Relatively easy


Tips and Pitfalls: In answering a question about resolving the discrepancy between two pieces of information, remember that the correct answer must explain both pieces of information. An answer that explains only one of the pieces of information is a tempting choice, but cannot be the correct answer to a question that asks you to resolve a discrepancy.



12. Gerrit: While browsing in a record store I noticed that one copy of a recording I wanted had mistakenly been priced at a quarter of the list price. When I finally reached the cashier I was told that the price had been mismarked and I would have to pay the full list price. Since I had wasted an hour standing in line, the retailer was morally obligated to sell me the recording at the lower price.


Saskia: I disagree. You knew that a mistake had been made, and you were simply trying to take advantage of that mistake.


Which one of the following principles, if established, would most help to justify Saskia's position?


(A) The price displayed on an item in a retail store morally constitutes an irrevocable offer to sell the item at that price.


(B) Customers of retail stores are morally entitled to profit from any mistakes that the retailers make in marking prices.


(C) Retailers are morally entitled to update marked prices periodically in order to reflect changes in manufacturers' suggested prices.


(D) Retailers are morally obligated to meet expectations about prices that they have intentionally encouraged their customers to hold.


(E) Retailers are morally obligated to sell an item to a customer at a mismarked price only if that customer was genuinely misled about the intended price by the mismarking.



General Description: This question asks you to find the principle that would most help justify Saskia's position in arguing against Gerrit. To do so, it is important that you understand exactly what Saskia's position is, and how it is supported.



A. Incorrect. This principle would actually help establish Gerrit's position, not Saskia's. If the principle in response (A) were established, it would follow that the retailer was morally obligated to sell Gerrit the recording at the lower, mistaken price, since that was the price actually marked on the recording.



B. Incorrect. This principle would actually help establish Gerrit's position, not Saskia's. If the principle in response (B) were established, it would follow that Gerrit was morally entitled to profit from the retailer's mistake, that is, that he was morally entitled to purchase the recording at the lower, incorrect price.



C. Incorrect. Since the conversation in the passage does not address the question of updating prices in response to changes in manufacturers' suggested prices, this principle cannot help justify Saskia's position.



D. Incorrect. Since the retailer discussed in the passage did not intentionally encourage Gerrit to hold a certain expectation about the price of the recording (Gerrit says the price was "mistakenly" marked), the establishment of this principle cannot help justify Saskia's position.



E. Correct. This principle presents a condition as necessary to create an obligation for a retailer to sell an item at a mismarked price: Only if the customer was "genuinely misled" about the price is such an obligation created. Since Gerrit was not genuinely misled about the price by the mismarking, the principle's necessary condition for such an obligation does not obtain in this case. That is, establishing the principle in response (E) would help justify Saskia's position that there is no such obligation in this case.



Difficulty Level: Easy


Tips and Pitfalls: To justify one side's position, it is not sufficient that a principle undermine the opposing side's position; the correct response must provide positive support for the position in question.



13. Treasure hunter: In general, archaeological artifacts found on public property cannot legally be privately owned. But according to centuries-old maritime law, people who risk their lives attempting to rescue a ship in peril are permitted to keep whatever cargo they can salvage. Under this rule treasure hunters clearly are entitled to keep the cargo from ancient shipwrecks that they risk their lives to save from oblivion in public waters.


Archaeologist: Not so. These shipwrecks have stabilized over the centuries they have lain underwater. The only danger they are in is from greedy treasure hunters who destroy archaeological evidence in their hurry to loot salable artifacts.


On the evidence of their statements, it can be concluded that the treasure hunter and the archaeologist disagree on which one of the following?


(A) what constitutes an archaeological artifact


(B) in what sense, if any, an ancient shipwreck can be said to be in peril


(C) whether treasure hunters risk their lives when they retrieve artifacts from ancient shipwrecks


(D) whether maritime law can ever be applied to a ship that has already sunk


(E) whether antique shipwrecks in public waters can properly be said to be on public property



General Description: This question asks you to identify an issue over which the treasure hunter and the archaeologist disagree. The task, then, is a matter of determining which of the choices is a claim supported by one speaker but rejected by the other.



A. Incorrect. Whether or not shipwrecks (or anything else, for that matter) count as archaeological artifacts does not arise in the passage.



B. Correct. The treasure hunter presents a rule applying to "a ship in peril" and claims that this rule applies also to the case of treasure hunters taking cargo from ancient shipwrecks. Thus the treasure hunter is saying that ancient shipwrecks count as "ships in peril" in the sense used in "centuries-old maritime law." The archaeologist, on the other hand, disagrees with this claim. By saying "these shipwrecks have stabilized over the centuries," the archaeologist shows disagreement with the claim that ancient shipwrecks are "ships in peril." By saying "The only danger they are in is from greedy treasure hunters," the archaeologist suggests a different sense in which ancient shipwrecks may be said to be in peril. Thus the treasure hunter and the archaeologist disagree over "in what sense, if any, an ancient shipwreck can be said to be- in peril."



C. Incorrect. The treasure hunter clearly believes that treasure hunters risk their lives in retrieving such artifacts, as shown by the reference in the last sentence to "cargo from ancient shipwrecks that they [i.e., treasure hunters] risk their lives to save." However, nothing the archaeologist says indicates disagreement with this claim. The archaeologist does make a claim about danger to the shipwrecks, but not about possible dangers to those who would retrieve their cargo. This was by far the most popular incorrect response.



D. Incorrect. The treasure hunter does bring maritime law into the discussion, but the archaeologist does not even address this issue in responding to the treasure hunter.



E. Incorrect. The treasure hunter brings into the discussion the general issue of the ownership of archaeological artifacts found on public property, but nothing the archaeologist says indicates any disagreement with the implication that antique shipwrecks can be said to be on public property.



Difficulty Level: Difficult


Tips and Pitfalls: In a "point of disagreement" question, a choice that presents a claim that is supported or denied by one speaker, but about which the other speaker has made no commitment either way, cannot be correct. In other words, for a response to be the correct response, it is not sufficient that the speakers might disagree about it. Limit your analysis to what is actually said; avoid the temptation to speculate about what (else) either speaker might believe.


14#
发表于 2006-3-19 19:26:00 | 只看该作者
谢谢
15#
发表于 2006-3-22 10:25:00 | 只看该作者
谢谢楼主
16#
发表于 2006-4-16 10:56:00 | 只看该作者
好好,咱也见识见识lsat OG
17#
发表于 2006-6-16 14:58:00 | 只看该作者
谢了

18#
发表于 2006-8-17 02:39:00 | 只看该作者

Thanks a million.

19#
发表于 2006-10-9 20:30:00 | 只看该作者
前人种树,后人乘凉
20#
发表于 2006-11-15 23:00:00 | 只看该作者

太谢谢楼主啦!真赞!

您需要登录后才可以回帖 登录 | 立即注册

Mark一下! 看一下! 顶楼主! 感谢分享! 快速回复:

所属分类: 法学院申请

近期活动

正在浏览此版块的会员 ()

手机版|ChaseDream|GMT+8, 2024-12-24 21:01
京公网安备11010202008513号 京ICP证101109号 京ICP备12012021号

ChaseDream 论坛

© 2003-2023 ChaseDream.com. All Rights Reserved.

返回顶部